1answer.
Ask question
Login Signup
Ask question
All categories
  • English
  • Mathematics
  • Social Studies
  • Business
  • History
  • Health
  • Geography
  • Biology
  • Physics
  • Chemistry
  • Computers and Technology
  • Arts
  • World Languages
  • Spanish
  • French
  • German
  • Advanced Placement (AP)
  • SAT
  • Medicine
  • Law
  • Engineering
uysha [10]
3 years ago
10

Which of the following equations is an example of the Associative Property of Addition?

Mathematics
2 answers:
Sholpan [36]3 years ago
8 0

Answer: I think the answer to your question is option B.

blagie [28]3 years ago
3 0

Answer: Choice B

(3+4) + 2 = 3 + (4+2)

=========================================

Explanation:

The general rule for the associative property of addition is

(a+b)+c = a+(b+c)

the a,b,c stay in the same order from left to right. The only thing that changes is the way we group the pair of terms. On the left side, we have a & b together with c left out. Then on the right side, we have b & c together with 'a' excluded.

Addition of any group of numbers can be done in any order, which is why this property is valid (though perhaps there's a more rigorous proof out there). For this specific problem, a = 3, b = 4, and c = 2.

You might be interested in
Find the area of a circle that has a diameter of 60 feet.
IrinaK [193]

Answer:

60π

Step-by-step explanation:

Hullo :D

I think this will be a piece of helpful info:

<em>The formula for finding the area of a circle if πd.</em>

<em />

In that case, we just substitute 60 (our d)

So the area will be 60π

If you want that multiplied, you'll get 188.4 (multiplying by 3.14)

Thus, the answer is $\boxed{60\pi}.

3 0
3 years ago
Is this right?<br> I’m not sure in my answer
gayaneshka [121]

Answer:

Yeah that seems good. ghg

5 0
3 years ago
I need help I do not understand this it's due tomorrow
Goshia [24]
I'll help you out with 3 questions.
For #1, you kind of figured it out yourself
That 90 degree angle on paper means that m/1 and m/2 make up 90
If m1 is 50, then just subtract m1 (50) from 90, making it 40. m/2 is 40

If Angle M4 is 45 degrees then angle M3 is 180-45 since they both make up a 180 degree angle.

There are 180 degrees in a triangle
If m 8 = 78
and m 9 = 60 
m 10 is 180 - 78 - 60


8 0
2 years ago
This is another problem for my math class<br> Solve the inequality below<br> X + 14 &gt; 27
frez [133]

Answer:

x = 14+

Step-by-step explanation:

If 14 + 14 = 28

Then

28 > 27 is true.

8 0
2 years ago
2 determinar el valor del patrimonio con los siguientes datos Caja= 10,000 Bancos= 13,500 Cuentas por pagar= 8,000 Documentos po
Softa [21]

Answer:

PN= $17,800

Step-by-step explanation:

<u>La formula general para calcular el activo de una empresa es la siguiente:</u>

<u></u>

A= PN + P

El activo es igual al patrimonio neto y el pasivo.

Debemos intercambiar los términos:

PN= A - P

Activo:

Caja= 10,000

Bancos= 13,500

Equipos de computación= 2,400

Documentos por cobrar= 5,100

Pasivo:

Cuentas por pagar= 8,000

Documentos por pagar= 4,000

Intereses por pagar= 1,200

PN= 31,000 - 13,200

PN= $17,800

7 0
3 years ago
Other questions:
  • Simplify the expression. 9n - 3n + 5.
    8·2 answers
  • Gwen plants two fifths of her garden in potatoes and one sixty of her garden in carrots. What fraction of the garden remains for
    11·1 answer
  • pediatricians prescribe 5 ml of cough syrup for every 25 lb of a child’s weight . How many millimeters of cough syrup will the d
    10·1 answer
  • Be first to answer! :)
    9·2 answers
  • A small bottle of lemonade is half the size of a large bottle. The small bottle costs 80p and the large bottle costs £1.50. Amir
    10·2 answers
  • 1. Check all solutions of the equation: 5x(x - 2) = 0
    11·1 answer
  • Which property does the statement illustrate? Column A Column B 1. (p + 3) + q = p + (3 + q) 2. 8 + (9 + 4) = (8 + 9) + 4 3. 4(x
    12·2 answers
  • PLease help will brainlist
    14·2 answers
  • Help....................................DDDDDDDDDDDDDDDDDDDDDDDDDDD
    12·1 answer
  • A swimming pool holds 12,500 gallons of water at the start of the day. A valve at the bottom of the pool releases water into a c
    7·1 answer
Add answer
Login
Not registered? Fast signup
Signup
Login Signup
Ask question!